If sin θ + cos θ = 2–√ cos (90°−θ), find cot θ.

Question: If $\sin \theta+\cos \theta=\sqrt{2} \cos \left(90^{\circ}-\theta\right)$, find $\cot \theta$. Solution: Given: $\sin \theta+\cos \theta=\sqrt{2} \cos \left(90^{\circ}-\theta\right)$ We have to find the value of $\cot \theta$. Now, $\sin \theta+\cos \theta=\sqrt{2} \cos \left(90^{\circ}-\theta\right)$ $\Rightarrow \sin \theta+\cos \theta=\sqrt{2} \sin \theta$ (since, $\cos \left(90^{\circ}-\theta\right)=\sin \theta$ ) $\Rightarrow \cos \theta=(\sqrt{2}-1) \sin \theta$ $\Rightarrow \fra...

Read More →

The product of two irrational number is

Question: The product of two irrational number is(a) always irrational(b) always rational(c) always an integer(d) sometimes rational and sometimes irrational Solution: (d) sometimes rational and sometimes irrational For example: $\sqrt{2}$ is an irrational number, when it is multiplied with itself it results into 2 , which is a rational number. $\sqrt{2}$ when multiplied with $\sqrt{3}$, which is also an irrational number, results into $\sqrt{6}$, which is an irrational number....

Read More →

An experiment succeeds twice as often as it fails.

Question: An experiment succeeds twice as often as it fails. Find the probability that in the next six trials, there will be at least 4 successes. Solution: The probability of success is twice the probability of failure. Let the probability of failure bex. Probability of success = 2x $x+2 x=1$ $\Rightarrow 3 x=1$ $\Rightarrow x=\frac{1}{3}$ $\therefore 2 x=\frac{2}{3}$ Let $p=\frac{1}{3}$ and $q=\frac{2}{3}$ Let X be the random variable that represents the number of successes in six trials. By b...

Read More →

Which of the following numbers is irrational?

Question: Which of the following numbers is irrational? (a) $\sqrt{\frac{4}{9}}$ (b) $\frac{\sqrt{1250}}{\sqrt{8}}$ (c) $\sqrt{8}$ (d) $\frac{\sqrt{24}}{\sqrt{6}}$ Solution: Since, $\sqrt{\frac{4}{9}}=\frac{2}{3}$, which is a rational number, $\frac{\sqrt{1250}}{\sqrt{8}}=\sqrt{\frac{1250}{8}}=\sqrt{\frac{625}{4}}=\frac{25}{2}$, which is a rational number, $\sqrt{8}=2 \sqrt{2}$, which is an irrational number, and $\frac{\sqrt{24}}{\sqrt{6}}=\sqrt{\frac{24}{6}}=\sqrt{4}=2$, which is a rational nu...

Read More →

Question: $\frac{1}{1.4}+\frac{1}{4.7}+\frac{1}{7.10}+\ldots+\frac{1}{(3 n-2)(3 n+1)}=\frac{n}{3 n+1}$ Solution: LetP(n) be the given statement. Now, $P(n)=\frac{1}{1.4}+\frac{1}{4.7}+\frac{1}{7.10}+\ldots+\frac{1}{(3 n-2)(3 n+1)}=\frac{n}{3 n+1}$ Step 1; $P(1)=\frac{1}{1.4}=\frac{1}{4}=\frac{1}{3 \times 1+1}$ Hence, $P(1)$ is true. Step 2: Let $P(m)$ be true. i.e., $\frac{1}{1.4}+\frac{1}{4.7}+\ldots+\frac{1}{(3 m-2)(3 m+1)}=\frac{m}{3 m+1}$ Now, $P(m)=\frac{1}{1.4}+\frac{1}{4.7}+\ldots+\frac{1...

Read More →

If a leap year is selected at random, what is the chance that it will contain 53 Tuesdays?

Question: If a leap year is selected at random, what is the chance that it will contain 53 Tuesdays? Solution: In a leap year, there are 366 days i.e., 52 weeks and 2 days. In 52 weeks, there are 52 Tuesdays. Therefore, the probability that the leap year will contain 53 Tuesdays is equal to the probability that the remaining 2 days will be Tuesdays. The remaining 2 days can be Monday and Tuesday Tuesday and Wednesday Wednesday and Thursday Thursday and Friday Friday and Saturday Saturday and Sun...

Read More →

How many digits are there in the repeating block of digits in the decimal expansion of

Question: How many digits are there in the repeating block of digits in the decimal expansion of $\frac{17}{7} ?$ (a) 16 (b) 6 (c) 26 (d) 7 Solution: $\because \frac{17}{7}=2 . \overline{428571}$ So, there are 6 digits in the repeating block of digits in the decimal expansion of $\frac{17}{7}$. Hence, the correct option is (b)....

Read More →

A die is thrown again and again until three sixes are obtained. Find the probability of obtaining the third six in the sixth throw of the die.

Question: A die is thrown again and again until three sixes are obtained. Find the probability of obtaining the third six in the sixth throw of the die. Solution: The probability of getting a six in a throw of die is $\frac{1}{6}$ and not getting a six is $\frac{5}{6}$. Let $p=\frac{1}{6}$ and $q=\frac{5}{6}$ The probability that the 2 sixes come in the first five throws of the die is ${ }^{5} C_{2}\left(\frac{1}{6}\right)^{2}\left(\frac{5}{6}\right)^{3}=\frac{10 \times(5)^{3}}{(6)^{5}}$ $\there...

Read More →

If cosec θ=1312, find the value of 2 sin θ−3 cos θ4 sin θ−9 cos θ.

Question: If $\operatorname{cosec} \theta=\frac{13}{12}$, find the value of $\frac{2 \sin \theta-3 \cos \theta}{4 \sin \theta-9 \cos \theta}$. Solution: Given: $\operatorname{cosec} \theta=\frac{13}{12}$ We have to find the value of the expression $\frac{2 \sin \theta-3 \cos \theta}{4 \sin \theta-9 \cos \theta}$. Now, $\operatorname{cosec} \theta=\frac{13}{12}$ $\Rightarrow \sin \theta=\frac{1}{\operatorname{cosec} \theta}=\frac{1}{\frac{13}{12}}=\frac{12}{13}$ $\cos \theta=\sqrt{1-\sin ^{2} \th...

Read More →

Which of the following is an irrational number?

Question: Which of the following is an irrational number? (a) $\sqrt{23}$ (b) $\sqrt{225}$ (c) $0.3799$ (d) $7 . \overline{478}$ Solution: Since, $\sqrt{225}=15$, which is an integer, 0.3799 is a number with terminating decimal expansion, and 7. $\overline{478}$ is a number with non-terminating recurring decimal expansion Also, 23 is a prime number. So, $\sqrt{23}$ is an irrational number. $\quad[\because \sqrt{n}$ is always an irrational number, if $n$ is a prime number. $]$ Hence, the correct ...

Read More →

Question: $\frac{1}{2.5}+\frac{1}{5.8}+\frac{1}{8.11}+\ldots+\frac{1}{(3 n-1)(3 n+2)}=\frac{n}{6 n+4}$ Solution: LetP(n) be the given statement. Now, $P(n)=\frac{1}{2.5}+\frac{1}{5.8}+\frac{1}{8.11}+\ldots+\frac{1}{(3 n-1)(3 n+2)}=\frac{n}{6 n+4}$ Step 1: $P(1)=\frac{1}{2.5}=\frac{1}{10}=\frac{1}{6+4}$ Hence, $P(1)$ is true. Step 2; Let $P(m)$ be true. Then, $\frac{1}{2.5}+\frac{1}{5.8}+\frac{1}{8.11}+\ldots+\frac{1}{(3 m-1)(3 m+2)}=\frac{m}{6 m+4}$ To prove : $P(m+1)$ is true. i. e., $\frac{1}{...

Read More →

If 3–√ tan θ=3 sin θ, find the value of sin2θ − cos2θ.

Question: If $\sqrt{3} \tan \theta=3 \sin \theta$, find the value of $\sin ^{2} \theta-\cos ^{2} \theta$. Solution: Given: $\sqrt{3} \tan \theta=3 \sin \theta$ We have to find the value of $\sin ^{2} \theta-\cos ^{2} \theta$. $\sqrt{3} \tan \theta=3 \sin \theta$ $\Rightarrow \sqrt{3} \frac{\sin \theta}{\cos \theta}=3 \sin \theta$ $\Rightarrow \cos \theta=\frac{\sqrt{3}}{3}=\frac{1}{\sqrt{3}}$ Therefore, $\sin ^{2} \theta-\cos ^{2} \theta=1-\cos ^{2} \theta-\cos ^{2} \theta \quad\left(\right.$ si...

Read More →

Decimal expansion of

Question: Decimal expansion of $\sqrt{2}$ is (a) a finite decimal(b) 1.4121(c) non-terminating recurring(d) non-terminating, non-recurring Solution: (c) a non-terminating and non-repeating decimal Because $\sqrt{2}$ is an irrational number, its decimal expansion is non-terminating and non-repeating....

Read More →

1 + 3 + 5 + ... + (2n − 1) =

Question: $1+3+5+\ldots+(2 n-1)=n^{2}$ i.e., the sum of first $n$ odd natural numbers is $n^{2}$. Solution: LetP(n) be the given statement. Now, $P(n)=1+3+5+\ldots+(2 n-1)=n^{2}$ Step 1: $P(1)=1=1^{2}$ Hence, $P(1)$ is true. Step 2; Let $P(m)$ be true. Then: $1+3+5+\ldots+(2 m-1)=m^{2}$ To prove : $P(m+1)$ is true. i. e., $1+3+5+\ldots+\{2(m+1)-1\}=(m+1)^{2}$ $\Rightarrow 1+3+5+\ldots+(2 m+1)=(m+1)^{2}$ Now, we have; $1+3+5+\ldots+(2 m-1)=m^{2}$ $\Rightarrow 1+3+\ldots+(2 m-1)+(2 m+1)=m^{2}+2 m+...

Read More →

Question: $\frac{1}{1.2}+\frac{1}{2.3}+\frac{1}{3.4}+\ldots+\frac{1}{n(n+1)}=\frac{n}{n+1}$ Solution: LetP(n) be the given statement. Now, $P(n)=\frac{1}{1.2}+\frac{1}{2.3}+\frac{1}{3.4}+\ldots+\frac{1}{n(n+1)}=\frac{n}{n+1}$ Step 1: $P(1)=\frac{1}{1.2}=\frac{1}{2}=\frac{1}{1+1}$ Hence, $P(1)$ is true. Step 2: Let $P(m)$ be true. Then, $\frac{1}{1.2}+\frac{1}{2.3}+\frac{1}{3.4}+\ldots+\frac{1}{m(m+1)}=\frac{m}{m+1}$ We shall now prove that $P(m+1)$ is true. i. e., $\frac{1}{1.2}+\frac{1}{2.3}+\f...

Read More →

1 + 3 + 3

Question: $1+3+3^{2}+\ldots+3^{n-1}=\frac{3^{n}-1}{2}$ Solution: LetP(n) be the given statement. Now, $P(n)=1+3+3^{2}+\ldots+3^{n-1}=\frac{3^{n}-1}{2}$ Step 1: $P(1)=1=\frac{3^{1}-1}{2}=\frac{2}{2}=1$ Hence, $P(1)$ is true. Step 2: Let $P(m)$ is true. then, $1+3+3^{2}+\ldots+3^{m-1}=\frac{3^{m}-1}{2}$ We shall prove that $P(m+1)$ is true. That is, $1+3+3^{2}+\ldots+3^{m}=\frac{3^{m+1}-1}{2}$ Now, we have: $1+3+3^{2}+\ldots+3^{m-1}=\frac{3^{m}-1}{2}$ $\Rightarrow 1+3+3^{2}+\ldots+3^{m-1}+3^{m}=\f...

Read More →

Solve this

Question: is(a) a rational number(b) an integer(c) an irrational number(d) a whole number Solution: Since, $\pi$ has a non-terminating non-recurring decimal expansion. So, $\pi$ is an irrational number. Hence, the correct option is (c)....

Read More →

Choose the rational number which does not lie between

Question: Choose the rational number which does not lie between $-\frac{2}{3}$ and $-\frac{1}{5}$. (a) $-\frac{3}{10}$ (b) $\frac{3}{10}$ (c) $-\frac{1}{4}$ (d) $-\frac{7}{20}$ Solution: We have, $-\frac{2}{3}=-\frac{2 \times 20}{3 \times 20}=-\frac{40}{60}$ and $-\frac{1}{5}=-\frac{1 \times 12}{5 \times 12}=-\frac{12}{60}$ And $,-\frac{3}{10}=-\frac{3 \times 6}{10 \times 6}=-\frac{18}{60}, \frac{3}{10}=\frac{3 \times 6}{10 \times 6}=\frac{18}{60},-\frac{1}{4}=-\frac{1 \times 15}{4 \times 15}=-\...

Read More →

If 3cosθ = 1, find the value of 6 sin2 θ+tan2 θ4 cos θ

Question: If $3 \cos \theta=1$, find the value of $\frac{6 \sin ^{2} \theta+\tan ^{2} \theta}{4 \cos \theta}$ Solution: Given: $3 \cos \theta=1$ We have to find the value of the expression $\frac{6 \sin ^{2} \theta+\tan ^{2} \theta}{4 \cos \theta}$ We have, $3 \cos \theta=1$ $\Rightarrow \cos \theta=\frac{1}{3}$ $\sin \theta=\sqrt{1-\cos ^{2} \theta}=\sqrt{1-\left(\frac{1}{3}\right)^{2}}=\frac{\sqrt{8}}{3}$ $\tan \theta=\frac{\sin \theta}{\cos \theta}=\frac{\frac{\sqrt{8}}{3}}{\frac{1}{3}}=\sqrt...

Read More →

12 + 22 + 32 + ... + n

Question: $1^{2}+2^{2}+3^{2}+\ldots+\mathrm{n}^{2}=\frac{n(n+1)(2 n+1)}{6}$ Solution: LetP(n) be the given statement. Now, $P(n)=1^{2}+2^{2}+3^{2}+\ldots+n^{2}=\frac{n(n+1)(2 n+1)}{6}$ Step 1 : $P(1)=1^{2}=\frac{1(1+1)(2+1)}{6}=\frac{6}{6}=1$ Hence, $P(1)$ is true. Step 2 : Let $P(m)$ be true. Then, $1^{2}+2^{2}+\ldots+m^{2}=\frac{m(m+1)(2 m+1)}{6}$ We shall now prove that $P(m+1)$ is true. i. e., $1^{2}+2^{2}+3^{2}+\ldots+(m+1)^{2}=\frac{(m+1)(m+2)(2 m+3)}{6}$ Now, $P(m)=1^{2}+2^{2}+3^{2}+\ldot...

Read More →

In a hurdle race, a player has to cross 10 hurdles.

Question: In a hurdle race, a player has to cross 10 hurdles. The probability that he will clear each hurdle is $\frac{5}{6}$. What is the probability that he will knock down fewer than 2 hurdles? Solution: Letpandqrespectively be the probabilities that the player will clear and knock down the hurdle. $\therefore p=\frac{5}{6}$ $\Rightarrow q=1-p=1-\frac{5}{6}=\frac{1}{6}$ Let X be the random variable that represents the number of times the player will knock down the hurdle. Therefore, by binomi...

Read More →

A rational number equivalent to

Question: A rational number equivalent to $\frac{7}{19}$ is (a) $\frac{17}{119}$ (b) $\frac{14}{57}$ (c) $\frac{21}{38}$ (d) $\frac{21}{57}$ Solution: Since, $\frac{7}{19}=\frac{7 \times 3}{19 \times 3}=\frac{21}{57}$ Hence, the correct option is (d)....

Read More →

If cot θ=3–√, find the value of cosec2 θ+cot2 θcosec2 θ−sec2 θ.

Question: If $\cot \theta=\sqrt{3}$, find the value of $\frac{\cos e c^{2} \theta+\cot ^{2} \theta}{\operatorname{cosec}^{2} \theta-\sec ^{2} \theta}$. Solution: Given: $\cot \theta=\sqrt{3}$ We have to find the value of the expression $\frac{\operatorname{cosec}^{2} \theta+\cot ^{2} \theta}{\operatorname{cosec}^{2} \theta-\sec ^{2} \theta}$. We know that, $\cot \theta=\sqrt{3} \Rightarrow \cot ^{2} \theta=3$ $\operatorname{cosec}^{2} \theta=1+\cot ^{2} \theta=1+(\sqrt{3})^{2}=4$ $\sec ^{2} \the...

Read More →

1 + 2 + 3 + ... + n =

Question: $1+2+3+\ldots+n=\frac{n(n+1)}{2}$ i.e. the sum of the first $n$ natural numbers is $\frac{n(n+1)}{2}$ Solution: Let P(n) be the given statement. Now, $\mathrm{P}(n)=1+2+3+\ldots+n=\frac{n(n+1)}{2}$ Step1: $P(1)=1=\frac{1(1+1)}{2}=1$ Hence, $P(1)$ is true. Step 2: Let $P(m)$ be true. Then, $1+2+3+\ldots+m=\frac{m(m+1)}{2}$ We shall now prove that $P(m+1)$ is true. i. e., $1+2+\ldots+(m+1)=\frac{(m+1)(m+2)}{2}$ Now, $1+2+\ldots+m=\frac{m(m+1)}{2}$ $\Rightarrow 1+2+\ldots m+m+1=\frac{m(m+...

Read More →

If cosec A=2–√, find the value of 2 sin2 A+3 cot2 A4(tan2 A−cos2 A)

Question: If $\operatorname{cosec} A=\sqrt{2}$, find the value of $\frac{2 \sin ^{2} A+3 \cot ^{2} A}{4\left(\tan ^{2} A-\cos ^{2} A\right)}$. Solution: Given: $\operatorname{cosec} A=\sqrt{2}$ We have to find the value of the expression $\frac{2 \sin ^{2} A+3 \cot ^{2} A}{4\left(\tan ^{2} A-\cos ^{2} A\right)}$ We know that, $\operatorname{cosec} A=\sqrt{2}$ $\Rightarrow \sin A=\frac{1}{\operatorname{cosec} A}=\frac{1}{\sqrt{2}}$ $\cos A=\sqrt{1-\sin ^{2} A}=\sqrt{1-\left(\frac{1}{\sqrt{2}}\rig...

Read More →